Inscription / Connexion Nouveau Sujet
Niveau exercices
Partager :

Fermat n =3

Posté par
Sylvieg Moderateur
24-06-23 à 17:20

Bonjour,
Pendant l'été où les demandes d?aide diminuent, j'ai l'intention de ressortir des sujets non aboutis.
C'est-à-dire où une solution satisfaisante n'a pas été donnée.

Je commence avec Fermat-Wiles n=3 - Tait - 1870
Il s'agit de démontrer que l'équation x3 + y3 = z3 n'a pas de solution dans les entiers non nuls, en utilisant cette propriété :
Si (x;y;z) est un triplet solution, alors (y(x^3+z^3) ; z(x^3-y^3) ; x(y^3+z^3)) est aussi un triplet solution.

* Modération > message édité le 2 juillet *

Posté par
fabo34
re : Fermat n =3 01-07-23 à 08:57

Bonjour Sylvieg

Bon. Ca n'a pas l'air t'intéresser les foules. Ou alors la solution n'est vraiment pas si "easy"?

J'aimerais juste ajouter ici ma découverte récente (que vous devez surement déjà connaître, mais un petit rappel ne fait jamais de mal): l'existence de propositions héréditaires non initialisables. Peut-être sommes-nous dans ce cas ici? (ce que je ne souhaite pas . du moins Tait l'aurait-il mentionné ?)

En effet, l'exemple suivant: P(n): "7 est multiple de 32n+4 - 2n "

P(n) est fausse, et pourtant elle est héréditaire! Quelle magnifique étrangeté !

Connaissez-vous d'autres propositions de ce type?

Posté par
carpediem
re : Fermat n =3 01-07-23 à 09:31

salut

ben oui il y en a une infinité et en particulier des exemples de divisibilité comme le tien :

Congruence : 10^n \pm 1  est multiple de 3 (de 9)

Démontrer par l'absurde qu'une proposition est fausse pour tt n

entre autre ...

Posté par
fabo34
re : Fermat n =3 01-07-23 à 16:32

merci carpediem
Ton exemple est beaucoup plus simple!

Sur la fonction de Tait, je n'arrive pas à me défaire de l'idée suivante: si (x,y,z) est un triplet solution, alors par symétrie (y,x,z) l'est aussi aussi. Or dans tous les triplets solutions, (x,y) ont forcément des parités différentes et z est forcément impair. Ainsi, un des 2 triplets images, de (x,y,z) ou de (y,x,z), donne un "z" pair, ce qui est impossible. Donc il y a un problème de symétrie; ça ne peut pas marcher!?

Qu'en pensez-vous?

Posté par
carpediem
re : Fermat n =3 01-07-23 à 20:36

oui je n'étais pas intervenu car je ne voyais rien d'intéressant ...

mais ton idée pr contre semble intéressante ... mais il y a tout de même un pb :

si (x, y, z) est un triplet solution il en est de même du triplet (kx, ky, kz) pour tout entier k

ce qui montre qu'avec k pair alors z l'est ...

en fait  en considérant les propositions :
P : (x, y, z) est solution
Q : (kx, ky, kz) est solution

alors s'il n'y a réellement pas de solution ce que j'ai écrit se traduit par P Q

et cette proposition implication est vrai puisque Faux Faux est vrai


maintenant on peut éliminer ce pb en considérant x et y premiers entre eux ce qui permet d'annuler mon argument ...

Posté par
fabo34
re : Fermat n =3 02-07-23 à 10:22

"ce qui montre qu'avec k pair alors z l'est ..."

Je ne crois pas tout saisir.
Si on a un triplet solution, ne peut-on pas supposer qu'il est primitif, et qu'il a nécessairement la forme (x,2y,z) ou (2y,x,z), avec x et z impair?

Posté par
Sylvieg Moderateur
re : Fermat n =3 02-07-23 à 13:29

Bonjour,
Je crois comprendre que c'est cette affirmation que carpediem remet en cause

Citation :
Or dans tous les triplets solutions, (x,y) ont forcément des parités différentes et z est forcément impair.
On peut partir d'un triplet où le pgcd est 1 ; mais les triplets que l'on peut en déduire peuvent ne pas avoir un pgcd égal à 1.

Posté par
fabo34
re : Fermat n =3 02-07-23 à 14:07

Merci.

Peut-être aurais-je dû écrire les triplets primitifs solutions? Cela dit, si on s'intéresse uniquement à la parité, la fonction de Tait n'assure-elle pas qu'en partant d'un triplet primitif, le pgcd du triplet image est différent de 2? Pourquoi supposer k=2, puisque ça ne peut pas arriver? Dans Fermat, ne recherche-t-on pas uniquement l'existence des triplets primitifs? La contradiction sur la parité du triplet image n'est-elle pas suffisante pour invalider l'existence d'un quelconque triplet primitif solution?

Est-ce que si on regarde le triplet image, un problème serait d'avoir un pgcd=k, k≠2, et après "réduction" se retrouver sur le même triplet. Faut-il et peut-on montrer facilement que ça n'est pas le cas?

Posté par
Sylvieg Moderateur
re : Fermat n =3 02-07-23 à 15:43

Dans Fermat, on cherche tous les triplets d'entiers naturels non nuls tels que x3 + y3 = z3.
Sans imposer qu'ils soient primitifs.
Cependant, s'il existait un triplet solution, on pourrait en déduire un triplet primitif solution.

Tes messages, fabo34, me permettent de voir une erreur dans mon énoncé.
Je vais demander à un modérateur dévoué de le rectifier

Posté par
fabo34
re : Fermat n =3 02-07-23 à 16:10

"s'il existait un triplet solution, on pourrait en déduire un triplet primitif solution" : Oui. merci. C'est ce que je voulais dire.

Et une fois précisé cela, je ne vois toujours pas ce qui cloche. Désolé. C'est dur de prendre du recul quand on a une idée fixe. Pourriez-vous me réexpliquer ce qui ne va pas: On part de l'existence d'un triplet primitif, donc (pair, impair, impair). Puis on en déduit l'existence d'un autre triplet absurde du type (impair, impair, pair). Que manque-t-il pour aller au bout?

Je vous jure que je fais des efforts

Posté par
Sylvieg Moderateur
re : Fermat n =3 02-07-23 à 16:50

Pourquoi ne pourrait-on pas avoir x et y impairs ?

Posté par
fabo34
re : Fermat n =3 02-07-23 à 17:22

Parce que j'ai complètement craqué. Et que je ne lisais pas la bonne ligne dans la table de modulo 8.

^1 1 2 3 4 5 6 7
^2 1 4 1 0 1 4 1
^3 1 0 3 0 5 0 7
^4 1 0 1 0 1 0 1
^5 1 0 3 0 5 0 7
^6 1 0 1 0 1 0 1
^7 1 0 3 0 5 0 7

Je suis vraiment désolé. C'est nul. Tout s'écroule.
Merci!

Reset pour les suivants. Mais je crois que j'ai bien pollué la page

Posté par
Sylvieg Moderateur
re : Fermat n =3 02-07-23 à 17:44

Pas de problème
C'est toi qui a lancé ce sujet intéressant en premier.
Il nous arrive à tous de faire des erreurs. Voir la fin de pgcd de trois entiers bis

Posté par
fabo34
re : Fermat n =3 02-07-23 à 17:57

L'essentiel est d'aider à sortir les gens de leurs "obsessions". En maths, ça peut vite prendre le dessus. Merci à toi et carpediem!
J'en profite pour vous remercier d'être là, tous les gens de ce site avec lesquel j'apprends tant de chose. En souhaitant de bonnes vacances à tous les profs!

La question de Taït reste toujours ouverte!

Posté par
fabo34
re : Fermat n =3 02-07-23 à 19:15

Et je viens de me souvenir d'où vient ma marotte.
J'avais lu ceci l'an dernier: https://www.researchgate.net/publication/316145391_A_special_case_of_Fermat's_last_theorem_and_it's_proof_by_combinatorics

Dans ce papier, il dit prouver que le cas impairn+impairn=pairn est impossible pour n⩾2

A l'époque, je m'étais dit que c'était évident avec le modulo 8, mais j'avais déjà mal mémorisé la table!

Si ça vous tente. Ca a l'air compliqué. Moi je vais regarder cela à mes heures perdues. En tout cas remplacer n par 3. Mais je me suis déjà fait tant avoir par des publis bidon sur researchgate !!

Peut-être que ça peut relancer le sujet?

Posté par
fabo34
re : Fermat n =3 04-07-23 à 20:05

Puisqu'il faut battre le fer tant qu'il est chaud, et bien cette publi est (encore) bidon. Les calculs sont faux pratiquement dès le départ.
Moi qui pensais que de rester dans  ℕ plutôt que dans ℤ pouvait apporter d'autres contraintes que la possibilité de descente infinie.

Bref: le mystère "Tait" reste ouvert. Pour l'instant, l'hypothèse la plus probable d'une indication "preuve easy" est un clin d'œil à la phrase "marge trop petite" .

A moins que quelqu'un ait une nouvelle piste ... en sachant que ça date de 1870

Posté par
carpediem
re : Fermat n =3 05-07-23 à 08:59

il me semble qu'on peut ne travailler que dans N :

si x^3 + y^3 = z^3 et :

x, y et z sont négatifs alors (-x, -y, -z) est solution et x, y et z sont positifs

x et z positifs et y < 0 alors (-y, z, x) est solution et -y, z et x sont positifs

x > 0 et y et z négatifs alors (x, -z, -y) est solution et x, -y, -z sont positifs

les autres cas s'obtiennent par multiplication par -1 de l'égalité et du fait que la fonction cube est impaire


mais bon ça ne fait guère avancer le schmilblick !!

Posté par
elhor_abdelali Correcteur
re : Fermat n =3 08-07-23 à 17:44

Bonjour

ce n'est pas vraiment une piste de résolution mais seulement une forme équivalente de l'équation

\Large\boxed{x^3+y^3=z^3\Longleftrightarrow(x+y-z)^3=3(x+y)(z-x)(z-y)} sauf erreur de ma part bien entendu

Posté par
fabo34
re : Fermat n =3 13-07-23 à 16:56

@carpediem: effectivement, je n'avais jamais vu les choses sous cet angle. ℕ  ou ℤ, même combat!

@elhor_abdelali: superbe relation. Et quelle factorisation! En sachant que ou x ou y ou z est forcément multiple de 3, je pensais voir une absurdité à gauche et à droite au niveau de la multiplicité sur le facteur 3, mais en fait non. J'avais déjà tourné en rond de la même manière avec les polynômes de Girard.

Et en parlant de factorisation, je rappelle celle qui permet de montrer la relation de Tait:

  y(x+z)³-x(y+z)³+z(x-y)³=(x + z) (x - y) (y + z) (x + y - z)

A partir de cela, en tâtonnant, je suis tombé sur celle-ci:

  (y(x+z))³+(-x(y+z))³+(z(x-y))³ = -3x y z (x + z) (x - y) (y + z)

Retour dans ℤ pour Fermat: on voit alors une "paramétrisation" de triplets non solutions, à savoir les triplets  (y(x+z), -x(y+z), z(x-y))_{x,y,z}.  Le membre de droite n'est alors jamais nul. Sauf pour x=y, z=-x,z=-y, ce qui donnent à chaque fois le triplet trivial (-k,0,k)_{k in {\mathbb {N}}

Ceci m'inspire cela: Cette paramétrisation peut-elle reconstituer ℤ³, et ainsi montrer qu'aucun triplet ne peut être solution? Des idées de votre côté sur cette forme?

Posté par
fabo34
re : Fermat n =3 14-07-23 à 08:30

Ce matin j'ai décidé que j'allais reformuler comme ceci. Moi aussi je veux mon nom dans la postérité  



(y(z-x))³+(x(y+z))³=(z(x+y))³+3xyz(x + y)(x - z)(y + z)

This furnishes an easy proof of the impossibility of finding two integers the sum of whose cubes is a cube.
Fabo 2023.



A vous pour la preuve ! ...

Posté par
fabo34
re : Fermat n =3 14-07-23 à 14:24

A mon avis, pas la peine de s'époumoner avec ça:
puisque ça donne pour n=2:

(y(z-x))^2+(x(y+z))^2=(z(x+y))^2+2xy(x - z)(y + z)

Du coup, c'est comme si on voulait utiliser cette relation pour prouver l'inexistence de triplet pythagoriciens!

Bref, la relation de Tait est une bizarrerie magnifique, mais ça ne doit pas prouver grand chose dans la généralité!
Donc peut-être pas la peine de se prendre la Tait !:D

Au passage, il semble qu'on ait systématiquement, avec n premier impair,

y(z-x))^n+(x(y+z))^n=(z(x+y))^n+n\times xyz(x+y)(x-z)(y+z)P(x,y,z)

Peut-être qu'au temps de Fermat, ils avaient l'habitude de manipuler des polynôme à 3 variables. Et que ce genre de factorisation leur était familières. Ça ne serait pas étonnant, Girard découvrait les relations avec les polynômes symétriques.

Et peut-être un raccourci trompeur de Fermat pour postuler son grand théorème? Le mystère reste entier. Ça serait quand même un peu gros comme bévue.

Un grand merci à Geogebra  et son module de calcul formel!
Vos remarques les bienvenues.

Posté par
fabo34
re : Fermat n =3 15-07-23 à 17:34

Bon. C'est vraiment nul ce que j'ai écrit précédemment, puisque c'est juste le développement de Newton de
( (a+b)+(c-b) )^3=(a+b)^3+(c-b)^3 +3(a+b)(c-b)(a+c)
Voilà. Sans intérêt aucun

Retour à Tait:

Pour me rattraper, j'ai trouvé plus intéressant ici: https://mathoverflow.net/questions/87293/on-a-remark-of-tait-on-flt-for-the-exponent-3  

Et tout le monde sèche! Problème irrésolu depuis des lustres, alors même qu'il a été publié dans des grandes revues ( American Mathematical Monthly )

Donc l'hypothèse "Tait et Fermat du même acabit" se renforce

Posté par
fabo34
re : Fermat n =3 01-08-23 à 10:24

Bonjour,

   je repensais à cela. Comment prouver "astucieusement" la relation suivante (trouvée avec un logiciel)?

  y(x+z)³-x(y+z)³+z(x-y)³=(x + z) (x - y) (y + z) (x + y - z)

J'essaie le raisonnement suivant: Considérer qu'à gauche et à droite ce sont des polynômes en  x. Étant  de degré 3, il suffirait alors de trouver 4 points communs pour qu'ils soient égaux.
On remarque 3 valeurs "immédiates": x=\left\{y,z,0\right\}. Mais il m'en manque 1. Ce qui obligerait quand même à "développer", et perdre le côté astucieux.

Si je résume, on pose:

P(x)=y(x+z)³-x(y+z)³+z(x-y)³
Q(x)=(x + z) (x - y) (y + z) (x + y - z)

On remarque qu'on a pour tout (y,z):
P(y)=0=Q(y)
P(z)=0=Q(z)
P(0)=yz^3-zy^3=yz(z²-y²) = Q(0)

P et Q coïncident en 3 points. Manque le 4ème facilement calculable. Je ne vois pas. Ou peut-être une autre méthode ?
Qu'en pensez-vous?

Posté par
fabo34
re : Fermat n =3 01-08-23 à 10:34

Petite coquille: c'est x=\left\{y,-z,0\right\}.

Posté par
fabo34
re : Fermat n =3 01-08-23 à 11:43

Je l'ai!   x=\frac{y-z}{2}

Facile à calculer, pas besoin de développer:

P(x)=\frac{3}{8} (z-y)(y+z)^3 = Q(x)

Conclusion: de degré 3, P(x) et Q(x) coïncident en x=\left\{y,-z,0,\frac{y-z}{2}\right\}. Ils sont donc égaux.

Sous votre contrôle?
Peut-être une autre méthode?

Posté par
Sylvieg Moderateur
re : Fermat n =3 03-08-23 à 10:16

C'est tout bon !

Posté par
fabo34
re : Fermat n =3 04-08-23 à 09:12

Merci Sylvie.
J'en profite ici pour compléter avec les autres puissances. Après les cubes, ça ne semble plus se goupiller aussi bien.  Le carré est magnifique!

  y(x+z)³-x(y+z)³+z(x-y)³=(x + z) (x - y) (y + z) (x + y - z)
  y(x+z)²-x(y+z)²+z(x-y)²=(x - z) (x - y) (y + z)
  y(x+z)^1-x(y+z)^1+z(x-y)^1=0

Posté par
fabo34
re : Fermat n =3 06-08-23 à 11:52

Juste pour illustrer la "proposition" de Tait.
Ici sur le problème a^3+b^3=c^2

En 1911, un certain "Gérardin" donne cette relation incroyable:

(x^3+4 y^3)^3 -(3 x^2 y)^3 = (x^3 +y^3 )(8 y^3 -x^3)^2

Même "récurrence": un triplet solution (x,y,z) en génère un autre  (x^3+4 y^3, -3 x^2 y,z(8 y^3 -x^3 ) ) .

Ici, ça fonctionne, car c'est "initialisable":  (1,2,3) est solution. Le suivant donne ( 33,-6,189). On ne tombe pas forcément sur un triplet "primitif". Pourtant il en existe, comme (11,37,228).
Ici si on "primitive", ça donne 11^3-2^3=3^3\times 7^2

Posté par
dpi
re : Fermat n =3 06-08-23 à 19:01

Bonsoir,
J'ai suivi de loin...
A noter quelques n³-p³=m²

8³-7³=13²
10³-6³=28²
14³-7³=49²
28³-7³=147²
32³-28³=104²
33³-6³=189²
40³-24³=224²
105³-104³=181²


³

Posté par
dpi
re : Fermat n =3 07-08-23 à 08:40

Et quelques n³+p³=m²

2³+1³=3²
2³+2³=4²
4³+8³=24²
7³+21³=98²
8³+8³=32²
9³+18³=81²
11³+37³=228²
16³+32³=192³
18³+18³=108²
22³+26³=168²

Posté par
fabo34
re : Fermat n =3 08-08-23 à 16:22

Merci dpi . En fait, il s'agissait juste d'essayer d'illustrer la méthode de Tait à un autre problème. Évidemment, ça fonctionne pour montrer l'existence d'une infinité de solution, pas pour l'inexistence!!? Dans la même veine, j'aurais pu prendre comme exemple les triplets pythagoriciens. (a,b,c) solution, alors (a^2-b^2,2ab,c^2) aussi. Mais ça date d'avant 1870! Et celle de Gérardin est incroyable.

Retour au problème x^3+y^3=z^3

Car justement, j'ai trouvé cette formule sur un site:   a(a+2b)^3-b(2a+b)^3=(a+b)(a-b)^3

Ça donne (a,b,c) solution alors (b(2a^3+b^3), c(a^3-b^3), a(a^3+2b^3) )
soit la transformation:
     (x,y,z)\rightarrow (y(2x^3+y^3), z(x^3-y^3), x(x^3+2y^3) ) aussi
Alors que celle de Tait est:
    (x;y;z)\rightarrow (y(x^3+z^3) ; z(x^3-y^3) ; x(y^3+z^3))  aussi

La récurrence de Tait n'est donc pas forcément unique, mais spéciale: elle utilise dans chacun nouveau membre les (x,y,z). Est-ce la clé? Que faire de ça?

Posté par
fabo34
re : Fermat n =3 08-08-23 à 19:37

Pfff ... j'écris des sornettes, (mais c'est en les écrivant que je m'en rends compte). Désolé.
x^3+z^3=2x^3+y^3 vu que x^3+y^3=z^3 
 \\
Donc tout ça c'est du pareil au même pareil.

Le truc à conserver, c'est qu'on arrive au résultat de Tait avec les relations suivantes (dont je trouve qu'il n'est pas aisé de faire le rapprochement):

a(a+2b)^3-b(2a+b)^3=(a+b)(a-b)^3

  ou

y(x+z)³-x(y+z)³+z(x-y)³=(x + z) (x - y) (y + z) (x + y - z)



Vous devez être membre accéder à ce service...

Pas encore inscrit ?

1 compte par personne, multi-compte interdit !

Ou identifiez-vous :


Rester sur la page

Désolé, votre version d'Internet Explorer est plus que périmée ! Merci de le mettre à jour ou de télécharger Firefox ou Google Chrome pour utiliser le site. Votre ordinateur vous remerciera !